Determining Coordinates - lines and planes

Click For Summary
To determine coordinates for lines and planes, it's essential to plug the obtained answers back into the original equations to verify they satisfy all conditions. A successful outcome indicates that the solution set is a single point, derived from having enough information to pinpoint specific values for each variable. If multiple points exist in the solution set, the final answer will express one or more variables in terms of others, limiting further resolution. This method ensures accuracy in solving for coordinates in geometric contexts. Verifying solutions against original equations is a critical step in confirming the correctness of the results.
ttpp1124
Messages
110
Reaction score
4
Homework Statement
Use normal vectors to determine the intersection, if any, for each of the following groups of three planes. Give a geometric interpretation in each case and the number of solutions for the corresponding linear system of equations. If the planes intersect in a line, determine a vector equation of the line. If the planes intersect in a point, determine the coordinates of the point.

I solved it, I just need someone to check my work. Thanks in advance!
Relevant Equations
n/a
IMG_3633.jpg
 
Physics news on Phys.org
You should plug the answers you got into the original equations and see if they satisfy all the equations.
 
FactChecker said:
You should plug the answers you got into the original equations and see if they satisfy all the equations.
Just did, and I got my desired outcome. Thanks!
 
To conclude that the solution set is a single point, you should notice that there was enough information to get one specific value for each variable. If there are more than one point in the solution set, your final answer will have one (or more) variable in terms of the others and you will not be able to go further.
 
Question: A clock's minute hand has length 4 and its hour hand has length 3. What is the distance between the tips at the moment when it is increasing most rapidly?(Putnam Exam Question) Answer: Making assumption that both the hands moves at constant angular velocities, the answer is ## \sqrt{7} .## But don't you think this assumption is somewhat doubtful and wrong?

Similar threads

  • · Replies 9 ·
Replies
9
Views
1K
Replies
1
Views
1K
  • · Replies 4 ·
Replies
4
Views
2K
  • · Replies 3 ·
Replies
3
Views
2K
Replies
2
Views
2K
  • · Replies 3 ·
Replies
3
Views
2K
Replies
7
Views
2K
  • · Replies 7 ·
Replies
7
Views
1K
  • · Replies 22 ·
Replies
22
Views
2K
Replies
7
Views
2K